BIMONTHY ASSESSMENT May 2021

Neha Pradeep

Roll no. 99

BIMONTHLY ASSESSMENT - MAY 2021

1) PULMONOLOGY:

Case #1:

https://soumyanadella128eloggm.blogspot.com/2021/05/a-55-year-old-female-with-shortness-of.html 

1) What is the evolution of the symptomatology in this patient in terms of an event timeline and where is the anatomical localization for the problem and what is the primary etiology of the patient's problem?

20 years ago

First SOB episode lasted one week.

It was relieved on taking medication.

19-13 years ago

Similar episodes, SOB lasted approximately one week. All episodes were relieved upon taking medication.

12 years ago

Episode of SOB, lasted 20 days.

She was hospitalized. SOB decreased upon treatment at the hospital.

11-9 years ago

SOB episode, lasting almost a month.

8 years ago

Diagnosed with diabetes. Presented with polyuria.

7-5 years ago

SOB episode lasted almost a month.


5 years ago

Treated for anemia with iron injections.

 

4-1 years ago

SOB episodes lasted almost a month.

 

30 days ago

Latest episode of SOB. SOB was insidious in onset and gradual in progression.

Initially SOB occurred on exertion and was relieved upon rest.

Generalized weakness.

20 days ago

Patient got HRCT done outside which showed signs of bronchiectasis.

Patient was diagnosed with hypertension.

 

15 days ago

Pedal edema up to ankle, pitting type.

Facial puffiness was present.

 

2 days ago

SOB at rest (grade 4) and was not relived with nebulizers. SOB was progressive. 

Patient experienced drowsiness. 

Decreased urine output was noticed. 

Anatomical Location of the problem: 

-       -This affects the respiratory system, such as the lungs and airways.

Primary Etiology:

-      - The main etiology for this patient could be linked to the patient’s occupation. Since the patient works at a paddy field this increases the chances of being exposed to fine dust and other particles that could get lodged in the respiratory passage. COPD can be found when there is prolonged exposure to these exogenous substances. This patient has findings of bronchiectasis.


-Reference: https://academic.oup.com/bmb/article/104/1/143/327395 

2) What are mechanism of action, indication and efficacy over placebo of each of the pharmacological and non pharmacological interventions used for this patient?

1. O2 inhalation:

-Since the patient's oxygen was at 75% at room air, so there was a requirement for supplemental oxygen.

-The indication of supplementation of oxygen therapy is when the oxygen levels are below 92% at room air the patient should be given supplementation. 

2. Intermittent BiPAP:

- Bilevel positive airway pressure (BiPAP) ventilation is a non invasive technique used to provide oxygen supplementation with those using a nasal mask. This is used spontaneously.

- MOA: This machine supplies pressurized air into the respiratory airways. This is an example of positive pressure ventilation. There are two settings with the pressure, one, low pressure for exhalation EPAP and two, for inhalation IPAP. 

- Indication: When a patient with COPD requires respiratory support, BiPAP machines are used.

3. Inj. Lasix IV BO if SBP is greater than 110 mmHg:

-It can be given to patients in two conditions: to treat hypertension and to relieve the symptoms of fluid retention in the body such as edema. 

-MOA: Luminal Na-K-Cl cotransporter which acts mainly in the thicker part in the ascending limb of the loop of Henle in the renal nephron, is acted upon by furosemide also called LASIK. This increases the excretion of the Na + and water by the kidneys. This in turns increases the urine output of the patient. 

4. NEB. with IPRAVENT, BUDECORT 6 hrly:

-Ipravent belongs to a group of medicines known as anticholinergic bronchodilators. They work by relaxing the bronchial passages. 

-Budecort belongs to a group of medicines called corticosteroids. It works by reducing and preventing swelling and inflammation in your lungs.

-Indication: Budesonide extended release capsules are indicated for the treatment and maintenance of mild to moderate Crohn’s disease. Various inhaled budesonide products are indicated for prophylactic therapy in asthma and reducing exacerbations of COPD. A budesonide nasal spray is available over the counter for symptoms of hay fever and upper respiratory allergies. Extended release capsules are indicated to induce remission of mild to moderate ulcerative colitis and a rectal foam is used for mild to moderate distal ulcerative colitis.

- MOA: The short term effects of corticosteroids are decreased vasodilation and permeability of capillaries, as well as decreased leukocyte migration to sites of inflammation.1 Corticosteroids binding to the glucocorticoid receptor mediates changes in gene expression that lead to multiple downstream effects over hours to days. Glucocorticoids inhibit neutrophil apoptosis and demargination; they inhibit phospholipase A2, which decreases the formation of arachidonic acid derivatives; they inhibit NF-Kappa B and other inflammatory transcription factors; they promote anti-inflammatory genes like interleukin-10. Lower doses of corticosteroids provide an anti-inflammatory effect, while higher doses are immunosuppressive. High doses of glucocorticoids for an extended period bind to the mineralocorticoid receptor, raising sodium levels and decreasing potassium levels. 

-Efficacy based on a study where Patients received 2 mg of budesonide every 6 h (n = 71),placebo (n = 66). All received standard treatment, including nebulized beta(2)-agonists, ipratropium bromide, oral antibiotics, and supplemental oxygen. The mean change (95% confidence interval) in postbronchodilator FEV(1) was greater with active treatments than with placebo: budesonide versus placebo, 0.10 L (0.02 to 0.18 L)

- Reference: https://go.drugbank.com/drugs/DB01222 

5. TAB PULMOCLEAR 100 mg PO OD:

- Pulmoclear Tablet is a combination of two mucolytic medicines: Acebrophylline and Acetylcysteine.

-It loosens mucus (phlegm) making it easier to cough. It also relaxes the airway muscles and thereby promotes easy inflow and outflow of air. 

6. Chest Physiotherapy:

- Chest physiotherapy improves lung function . Chest PT, or CPT expands the lungs, strengthens breathing muscles, loosens and improves drainage of thick lung secretions.

7. GRBS 6 hrly: 

 -This is used to monitor blood sugar levels. 

8. INJ. HAI SC:

-Human Actrapid Injection contains human insulin. It is given to the patient to lower blood sugar levels as she is a diabetic.

9. I/O charting:

- This is used to record fluid intake and output of the patient.

10. Head end elevation of 30-45°: 

- It is recommended in this patient since she is on mechanical ventilation to reduce the incidence of ventilator associated pneumonia that occurs due to aspiration of contaminated oropharyngeal secretions following endotracheal tube intubation.

- Efficacy based on studies: Moderate quality evidence from eight studies involving 759 participants demonstrated that a semi-recumbent (30º to 60º) position reduced clinically suspected VAP by 25.7% when compared to a 0° to 10° supine position. 

11. Injection Augmentin 1.2gm IV /BO:

 -It is given to the patient to treat bronchiectasis. 

 -Augmentin is a combination of Amoxicillin and Clavulanic Acid. The former binds to penicillin binding proteins in the cell wall of bacteria. It inhibits the synthesis of cell wall. The latter is a beta lactamase enzyme inhibitor, it facilitates action of amoxicillin.  

 -MOA: Amoxicillin competitively inhibits penicillin-binding protein 1 and other high molecular weight penicillin binding proteins. Penicillin bind proteins are responsible for glycosyltransferase and transpeptidase reactions that lead to cross-linking of D-alanine and D-aspartic acid in bacterial cell walls. Without the action of penicillin binding proteins, bacteria upregulate autolytic enzymes and are unable to build and repair the cell wall, leading to bactericidal action.

-Indication: Amoxicillin alone is indicated to treat susceptible bacterial infections of the ear, nose, throat, genitourinary tract, skin, skin structure, and lower respiratory tract. Amoxicillin is given with clavulanic acid to treat acute bacterial sinusitis, community acquired pneumonia, lower respiratory tract infections, acute bacterial otitis media, skin and skin structure infections, and urinary tract infections. Amoxicillin is given with omeprazole in the treatment of H. pylori.

-Reference: https://go.drugbank.com/drugs/DB01060 

12. Tab. Azithromycin 500mg OD

 -It is given to the patient to provide symptomatic relief and reduce incidence of acute exacerbations of COPD.

- Efficacy based on a study:  A randomized controlled trial found that patients hospitalized for an acute exacerbation of chronic obstructive pulmonary disease (COPD) experienced reduced rates of treatment failure when adding azithromycin to their standard of care. During the study, patients received a low dose of azithromycin in addition to their prescribed medications while in the hospital and continued taking the antibiotic for 3 months following hospitalization. The result, according to the data, was reduced treatment failure compared with standard of care alone. Rates of treatment failure were under 50% for patients taking azithromycin (49%) compared with 60% for patients receiving standard of care.

-MOA: To replicate, bacteria require a specific process of protein synthesis, enabled by ribosomal proteins. Azithromycin binds to the 23S rRNA of the bacterial 50S ribosomal subunit. It stops bacterial protein synthesis by inhibiting the transpeptidation/translocation step of protein synthesis and by inhibiting the assembly of the 50S ribosomal subunit. This results in the control of various bacterial infections. The strong affinity of macrolides, including azithromycin, for bacterial ribosomes, is consistent with their broadspectrum antibacterial activities.

-Indication: Azithromycin should be used only to treat or prevent infections that are proven or strongly suspected to be caused by susceptible bacteria to prevent the development antimicrobial resistance and maintain the efficacy of azithromycin.

 -Reference: https://go.drugbank.com/drugs/DB00207

13. INJ. HYDROCORTISONE 100 mg IV:

- It acts by reducing inflammation in the body. 

-Efficacy based on a study: Based on a study, in comparison to placebo, systemic corticosteroids can decrease the rate of failure of treatment and the risk of relapse along with improving the airflow. With the improvement of these symptoms there is a decreased length of the hospital stay. In this study, patients hospitalized with acute respiratory insufficiency and COPD were randomized to receive either intravenous (IV) corticosteroid (n=22) or matching placebo (n=22) for 72 hours. All patients received standardized treatment consisting of oxygen, aminophylline, nebulized isoproterenol, and antibiotics. The mean percentage change in both pre- and postbronchodilator forced expiratory volume in 1 second (FEV1) was significantly greater in patients receiving corticosteroid in comparison to placebo at all measured time points. 

-MOA: The short-term effects of corticosteroids are decreased vasodilation and permeability of capillaries, as well as decreased leukocyte migration to sites of inflammation. Corticosteroids binding to the glucocorticoid receptor mediates changes in gene expression that lead to multiple downstream effects over hours to days.

Glucocorticoids inhibit neutrophil apoptosis and demargination; they inhibit phospholipase A2, which decreases the formation of arachidonic acid derivatives; they inhibit NF-Kappa B and other inflammatory transcription factors; they promote anti-inflammatory genes like interleukin-10.

Lower doses of corticosteroids provide an anti-inflammatory effect, while higher doses are immunosuppressive. High doses of glucocorticoids for an extended period bind to the mineralocorticoid receptor, raising sodium levels and decreasing potassium levels.

14. INJ. THIAMINE 1 amp in 100 ml of NS:

-Efficacy based on a study:  The administration of a single dose of thiamine was associated with a trend toward increase in oxygen consumption in critically ill patients. Thiamine deficiency is seen in patients taking loop diuretics, as this patient is receiving LASIK, the use of thiamine could be prophylactic.

3) What could be the causes for her current acute exacerbation?

 - The main causes of exacerbation in normal cases of COPD are the interaction between the host, foreign organisms, and environment pollution. In the case of such a patient, she commonly had frequent episodes of exacerbation of shortness of breath. When the frequency of these episodes increases the rates of morbidity and mortality increase too. This leads to decrease lung function.

- In known cases of HTN, when there is an acute exacerbation of the underlying COPD, there is an elevation of the pulmonary pressure, which occurs along with the hypoxia present. This leads to right sided heart failure

- History of exacerbations, Diabetes, HTN – right heart failure

Reference: https://www.ncbi.nlm.nih.gov/pmc/articles/PMC2080749/#:~:text=Exacerbations%20of%20COPD%20are%20thought,defences%20leading%20to%20tissue%20damage. 

https://www.ncbi.nlm.nih.gov/pmc/articles/PMC4646156/#:~:text=In%20the%20acute%20exacerbation%20of,sided%20heart%20failure%20%5B28%5D.

4. Could the ATT have affected her symptoms? If so how?
-There are some case reports about interstitial lung disease (ILD) such as pneumonitis caused by isoniazid, rifampin, ethambutol. Anti-tubercular therapy can cause Acute kidney injury, which leads to pedal edema and facial puffiness. Therefore The causative drug was discontinued permanently or re-administrated after desensitization therapy.

5.What could be the causes for her electrolyte imbalance?
The distribution of electrolyte disturbances in COPD group (total 58.53%) was found as follows: hypokalemia in 20%, hyponatremia in 13.33%, hypomagnesemia in 6.66%, hypochloremia in 3.33%, and combined disturbances in 15%.

-Activation of the renin-angiotensin-aldosterone system and inappropriately elevated plasma arginine vasopressin in COPD may aggravate the electrolyte imbalance during acute exacerbation of COPD

-This patient has Hyponatremia and Hypochloremia according to the reports. 

-Hyponatremia in patients with COPD developed secondary to many reasons, such as development or worsening of hypoxia, hypercapnia, and respiratory acidosis, and right-side heart failure with development of lower limb edema, renal insufficiency, use of diuretics. This hyponatremia has risen due do the AKI present in the patient.

-The heart failure seen and the prescription of Lasix, has lead to hypochloremia. 

- Respiratory acidosis with metabolic alkalosis in patients with COPD with chronic hypercapnia is the usual cause of hypochloremia in those patients.

-Reference: https://www.ncbi.nlm.nih.gov/pmc/articles/PMC7544412/


2) NEUROLOGY: 

Case #1:

1) What is the evolution of the symptomatology in this patient in terms of an event timeline and where is the anatomical localization for the problem and what is the primary etiology of the patient's problem?

Patient has a history of seizures. The timeline of this patient is as follows:

 

1 year ago

First episode of seizure

 

4 months ago

Second episode of seizure (24 hours after with the withdrawal of alcohol, leading to restlessness, sweating and tremors)

 

9 days ago

Started talking and laughing to himself, decreased food intake, unable to recognize family members, has short term memory loss.

 


Anatomical localization: 

-There are lesions in the CNS.

Etiology:

- Since the Patient has a history of consumption of alcohol, this will lead to a deficiency in thiamine. A thiamine deficiency gives rise to Wernicke’s Encephalopathy.

Thiamine is a helper molecule required by three enzymes involved in two pathways of carbohydrate metabolism. Because intermediate products of these pathways are needed for the generation of other essential molecules in the cells (e.g., building blocks of proteins and DNA as well as brain chemicals), a reduction in thiamine can interfere with numerous cellular functions, leading to serious brain disorders, including Wernicke–Korsakoff syndrome, which is found predominantly in alcoholics. 





2) What are mechanism of action, indication and efficacy over placebo of each of the pharmacological and non pharmacological interventions used for this patient?

 

1.   1.IVF NS and RL @150ml/hr

- Normal saline and ringer lactate solutions are both crystalloid fluids. NS contains 154 mM Na+ and Cl-, with an average pH of 5.0 and osmolarity of 308 mOsm/L. LR solution has an average pH of 6.5, is hypo-osmolar (272 mOsm/L), and has similar electrolytes (130 mM Na+, 109 mM Cl-, 28 mM lactate, etc.) to plasma.

 

2.    2. Inj. 1amp THIAMINE in 100ml NS, TID

- Thiamine is given in patients that are chronic alcoholics, due to the pathology which causes the thiamine levels in the body are deficient. Thiamine is required in the breakdown of glucose.

- According to the given article, thiamine is necessary and is provides great treatment in cases of Wernickes Encephalopathy.

- https://www.ncbi.nlm.nih.gov/pmc/articles/PMC7163251/

3.    3. Inj. Lorazepam

-       Lorazepam is mostly given to reduce the anxiety the patient feels.

-       https://pubmed.ncbi.nlm.nih.gov/24646/

-       According to the trial given above, Lorazepam has a 50% better result rate in reducing the patient’s anxiety than a placebo. In this case Lorazepam was given at 3mg, b.i.d.

4.    4. T. Pregabalin 75mg/PO/ BD

-       MOA: Although the mechanism of action has not been fully elucidated, studies involving structurally related drugs suggest that presynaptic binding of pregabalin to voltage-gated calcium channels is key to the antiseizure and antinociceptive effects observed in animal models. By binding presynaptically to the alpha2-delta subunit of voltage-gated calcium channels in the central nervous system, pregabalin modulates the release of several excitatory neurotransmitters including glutamate, substance-P, norepinephrine, and calcitonin gene related peptide. In addition, pregabalin prevents the alpha2-delta subunit from being trafficked from the dorsal root ganglia to the spinal dorsal horn, which may also contribute to the mechanism of action. Although pregabalin is a structural derivative of the inhibitory neurotransmitter gamma-aminobutyric acid (GABA), it does not bind directly to GABA or benzodiazepine receptors.

-       Indication: Pregabalin is indicated for the management of neuropathic pain associated with diabetic peripheral neuropathy, postherpetic neuralgia, fibromyalgia, neuropathic pain associated with spinal cord injury, and as adjunctive therapy for the treatment of partial-onset seizures in patients 1 month of age and older.

5.    5. Lactulose 30ml/PO/BD

-       MOA: Lactulose is a synthetic disaccharide derivative of lactose that consists of one molecule of galactose and one molecule of fructose. Saccharolytic bacteria present in the large intestine subsequently break the substance down into organic acids like lactic acid and small amounts of formic and acetic acids. Such resultant volatile fatty acid metabolites, in combination with hydrogen and methane that is also generated consequently increase intraluminal gas formation, gut motility, and elicit an osmotic effect that facilitates an increase in the water content of stool as well as associated stool softening. All of these actions ultimately assist in facilitating and increasing the frequency of bowel movements in patients experiencing constipation, although it may take 24 to 48 hours after using the medication for this laxative effect to become evident. At the same time, the formation of such acids via the metabolism of lactulose by colonic bacteria also acidifies the contents of the colon, thereby contributing to the treatment of portal-systemic encephalopathy (PSE). As one of the principal features of PSE involves the accumulation of nitrogenous waste products like ammonia in the systemic circulation, a state in which the colonic contents become more acidic than blood allows ammonia in the circulation to diffuse into the colon.. Furthermore, ammonia that diffuses into the acidic colon is ionized to ammonium ions that are incapable of being absorbed back into the blood. These effects, combined And finally, it is also believed that an acidic colonic environment results in the elimination of urease-producing bacteria that contribute to the formation of ammonia while surviving colonic bacteria use up any trapped ammonia in the colon as a source of nitrogen for protein synthesis.4

-       Indication: Lactulose is also employed as an adjunct to protein restriction and supportive therapy for the prevention and treatment of portal-systemic encephalopathy (PSE), including both the hepatic pre-coma and coma variations In particular, lactulose solution has been effective at managing PSE resulting from surgical portacaval shunts or from chronic hepatic diseases like cirrhosis. Moreover, there have also been studies demonstrating the capacity for lactulose to minimize the formation of gallstones and even some investigations regarding the experimental use of the agent in developing novel anticancer agents owing to its ability to bind galactin carbohydrates involved in various tumor progressions.

7.   6. Inj 2 ampoule KCl (40mEq) in 10 NS over 4 hours

-       MOA: Supplemental potassium in the form of high potassium food or potassium chloride may be able to restore normal potassium levels.

-       Indication: For use as an electrolyte replenisher and in the treatment of hypokalemia.

     7.Syp. Potchlor 10ml in one glass water/PO/BD

-       It is a supplement of Potassium, it has the above given mechanism of action and indications.

 

3) Why have neurological symptoms appeared this time, that were absent during withdrawal earlier? What could be a possible cause for this?

-https://jnnp.bmj.com/content/75/suppl_3/iii16

- In this case, the patient mainly deals with two main neurological symptoms: seizure activity and memory loss.

a) Seizures: Repeated alcohol intake and withdrawal is termed as kindling. Kindling is a process whereby there can be small chemical and electrical stimuli which can precipitate the seizure activity.

b) Memory Loss:  Alcohol may have a direct neurotoxic effect on cortical neurons, but much of the damage may be secondary to a pre-existing pathology caused by thiamine deficiency. Studies have shown that Wernicke Encephalopathy patients have widespread cerebral and subcortical atrophy which is greater than that found in the alcoholic patients without amnesia.

-With the continuous use of alcohol, there is a constant decrease in the levels on thiamine in the body, which gives rise to delayed neurological symptoms.

 

4) What is the reason for giving thiamine in this patient?

-Thiamine is vastly given in patients with this presentation because of the known history of alcohol withdrawal and alcohol habituation.

-A chronic alcoholic has a depleted supply of thiamine in the body which can give rise to neurological symptoms, such as Wernicke Encephalopathy.

-To abate some of the symptoms, thiamine is given to replenish the supply of the patient.

Reference: https://www.ncbi.nlm.nih.gov/pmc/articles/PMC5354137/#:~:text=Wernicke's%20encephalopathy%20is%20an%20acute,nystagmus%20and%20ophthalmoplegia%20(1).

5) What is the probable reason for kidney injury in this patient? 

 


- https://www.ncbi.nlm.nih.gov/pmc/articles/PMC5513691/

- Mechanism for alcohol-induced kidney injury is: Chronic alcohol consumption induces profound injury in several organs that may affect and aggravate the effect of ethanol on the kidney. Ethanol itself markedly induces the expression of the microsomal ethanol oxidation system (CYP2E1), producing reactive oxygen species as a byproduct. Increased gastrointestinal permeability and endotoxin load may lead to alcoholic steatohepatitis resulting in excessive immunoglobulin A (IgA) load. IgA deposits may accumulate in the kidney, leading to glomerulopathy. Renal microcirculatory changes in advanced liver cirrhosis leads to hepatorenal syndrome. Alcohol-induced skeletal muscle damage leads to excessive amounts of circulating myoglobin, causing renal tubular injury because of increased oxidative stress. Due to the development of alcoholic cardiomyopathy, chronic renal hypoxia develops, activating the renin–angiotensin–aldosterone system (RAAS), which in turn leads to further free radical production and to the propagation of fibrotic pathways.

 

6). What is the probable cause for the normocytic anemia?

-The probable cause for normocytic anemia is kidney disease.

 

7) Could chronic alcoholism have aggravated the foot ulcer formation? If yes, how and why?

-Yes, chronic alcoholism could have aggravated the formation on the foot ulcer. In the case of a chronic alcoholic, there is a depression in the immune system, the same way it is for diabetics. This can also lead to alcoholic neuropathy. Alcoholic neuropathy involves coasting caused by damage to nerves that results from long term excessive drinking of alcohol and is characterized by spontaneous burning pain, hyperalgesia, and allodynia. Chronic presentation will increase the chances of foot ulcer formation and also increase the time of recovery.  

-Reference: https://americanaddictioncenters.org/alcoholism-treatment/ulcers



Case #2:

1) What is the evolution of the symptomology in this patient in terms of an event timeline and where is the anatomical localization for the problem and what is the primary etiology of the patients problem?

Timeline of the patient is:

7 days ago

Patient gave a history of giddiness that started around 7 in the morning; subsided upon taking rest; associated with one episode of vomiting

 

4 days ago

Patient consumed alcohol; He developed giddiness that was sudden onset, continuous and gradually progressive. It increased on standing and while walking.

 

History of

Postural instability (falls while walking)

Associated with

Bilateral hearing loss, aural fullness, presence of tinnitus.

Vomiting- 2-3 episodes per day, non projectile, non bilious without food particles

At admission

Slurring of speech, deviation of mouth that got resolved the same day

 

 

Anatomical location

-In the cerebellar hemisphere of the brain in the central nervous system, there is an infract present.

Etiology

-Ataxia is the lack of muscle control or co-ordination of voluntary movements, such as walking or picking up objects. This is usually a result of damage to the cerebellum (part of the brain that controls muscle co-ordination)

-Many conditions cause cerebellar ataxia- Head trauma, Alcohol abuse, certain medications eg. barbituates, stroke, tumors, cerebral palsy, brain degeneration etc.

-In this case, the patient has hypertension for which he has been prescribed medication that he has not taken. Stroke due to an infarct can be caused by blockade or bleeding in the brain due to which blood supply to the brain is decreased, depriving it of essential oxygen and nutrients. This process could’ve caused the infarct formation in the cerebellar region of the brain, thus causing cerebellar ataxia.

-Reference: https://www.scielo.br/j/anp/a/NKwNPtFnhDz3zWNh7Gvv7nd/?lang=en

2)  What are the mechanism of action, indication and efficacy over placebo of each of the pharmacological and non pharmacological interventions used for this patient?

1. Tab Vertin 8mg- 

- This is Betahistine, which is an anti- vertigo medication.

-MOA: Vertigo is a disturbing sensation of movement caused by dysfunction of the labyrinth (inner ear), vestibular nerve, cerebellum, brainstem, or Central Nervous System (CNS). Vestibular forms of vertigo are often accompanied by auditory dysfunctions such as hyperacusis, hearing loss, and tinnitus.3 In most cases, adaptive mechanisms of the CNS lead to functional recovery after episodes of vertigo, however, syndromes such as Ménière's disease tend to cause the recurrence of vertigo symptoms. This significantly impacts the quality of life and the ability to carry out daily activities.

H1-receptor activity:

The mechanism of action of Betahistine is multifactorial. Meniere's disease is thought to result from a disruption of endolymphatic fluid homeostasis in the ear. Betahistine mainly acts as a histamine H1-receptor agonist. The stimulation of H1-receptors in the inner ear causes a vasodilatory effect leading to increased permeability of blood vessels and a reduction in endolymphatic pressure; this action prevents the rupture of the labyrinth, which can contribute to the hearing loss associated with Meniere's disease. Betahistine is also purported to act by reducing the asymmetrical functioning of sensory vestibular organs and increasing vestibulocochlear blood flow, relieving symptoms of vertigo.

H3-receptor activity:

In addition to the above mechanisms, Betahistine also acts as a histamine H3-receptor antagonist, increasing the turnover of histamine from postsynaptic histaminergic nerve receptors, subsequently leading to an increase in H1-agonist activity. H3-receptor antagonism elevates levels of neurotransmitters including serotonin in the brainstem, inhibiting the activity of vestibular nuclei, thus restoring proper balance and decreasing vertigo symptoms. 

-Indications: Betahistine is indicated for the reduction of recurrent vertigo episodes associated with Ménière's disease in patients 18 years old and above. In patient's with a history of giddiness and balance issues. 

-Reference: https://go.drugbank.com/drugs/DB06698

2. Tab Zofer 4mg: 

-This is Ondansetron. It is an anti emetic. 

-MOA: Ondansetron is a selective antagonist of the serotonin receptor subtype, 5-HT3. Cytotoxic chemotherapy and radiotherapy are associated with the release of serotonin (5-HT) from enterochromaffin cells of the small intestine, presumably initiating a vomiting reflex through stimulation of 5-HT3 receptors located on vagal afferents. Ondansetron may block the initiation of this reflex. Activation of vagal afferents may also cause a central release of serotonin from the chemoreceptor trigger zone of the area postrema, located on the floor of the fourth ventricle. Thus, the antiemetic effect of ondansetron is probably due to the selective antagonism of 5-HT3 receptors on neurons located in either the peripheral or central nervous systems, or both. Although the mechanisms of action of ondansetron in treating postoperative nausea and vomiting and cytotoxic induced nausea and vomiting may share similar pathways, the role of ondansetron in opiate-induced emesis has not yet been formally established.

-Indications: In the adult patient population: 

i) Orally administered ondansetron tablets and orally disintegrating tablets (ODT) are indicated for: - the prevention of nausea and vomiting associated with emetogenic cancer chemotherapy, including high dose (ie. greater than or equal to 50 mg/m2) cisplatin therapy, and radiotherapy, and - the prevention and treatment of postoperative nausea and vomiting

ii) Intravenously administered ondansetron injection formulations are indicated for: - the prevention of nausea and vomiting associated with emetogenic cancer chemotherapy, including high dose (ie. greater than or equal to 50 mg/m2) cisplatin therapy, and - the prevention and treatment of postoperative nausea and vomiting 

3. Tab Ecosprin 75mg:

-This is aspirin. It is an NSAID. Also known as acetylsalicylic acid. 

-MOA-:Acetylsalicylic acid (ASA) blocks prostaglandin synthesis. It is non-selective for COX-1 and COX-2 enzymes. Inhibition of COX-1 results in the inhibition of platelet aggregation for about 7-10 days (average platelet lifespan). The acetyl group of acetylsalicylic acid binds with a serine residue of the cyclooxygenase-1 (COX-1) enzyme, leading to irreversible inhibition. This prevents the production of pain-causing prostaglandins. This process also stops the conversion of arachidonic acid to thromboxane A2 (TXA2), which is a potent inducer of platelet aggregation. Platelet aggregation can result in clots and harmful venous and arterial thromboembolism, leading to conditions such as pulmonary embolism and stroke. It is important to note that there is 60% homology between the protein structures of COX-1 and COX-2. ASA binds to serine 516 residue on the active site of COX-2 in the same fashion as its binding to the serine 530 residue located on the active site of COX-1. The active site of COX-2 is, however, slightly larger than the active site of COX-1, so that arachidonic acid (which later becomes prostaglandins) manages to bypass the aspirin molecule inactivating COX-2. ASA, therefore, exerts more action on the COX-1 receptor rather than on the COX-2 receptor. A higher dose of acetylsalicylic acid is required for COX-2 inhibition.

-Indications: ASA is also indicated for various other purposes, due to its ability to inhibit platelet aggregation. These include. Reducing the risk of cardiovascular death in suspected cases of myocardial infarction (MI). Reducing the risk of a first non-fatal myocardial infarction in patients, and for reducing the risk of morbidity and mortality in cases of unstable angina and in those who have had a prior myocardial infarction. For reducing the risk of transient ischemic attacks (TIA) and to prevent atherothrombotic cerebral infarction (in conjunction with other treatments). For the prevention of thromboembolism after hip replacement surgery. For decreasing platelet to platelet adhesion following carotid endarterectomy, aiding in the prevention of transient ischemic attacks (TIA). Used for patients undergoing hemodialysis with a silicone rubber arteriovenous cannula inserted to prevent thrombosis at the insertion site.

4.  Tab Atorvostatin 40mg:

-This is a statin

-MOA: It is an HMG CoA reductase inhibitor and thus inhibits the rate limiting step in cholesterol biosynthesis. It decreases blood LDL and VLDL, decreases cholesterol synthesis, thus increasing LDL receptors in liver and increasing LDL uptake and degeneration. Hence plasma LDL level decreases.

-Indications: Used to treat primary hyperlipidemias. In this case it is used for primary prevention of stroke.

5. Clopidogrel 75mg: 

-It is an antiplatelet medication.

-MOA: It inhibits ADP mediated platelet aggregation by blocking P2Y12 receptor on the platelets.

-Indications: In this case it decreases the risk of heart disease and stroke by preventing clotting

6. Thiamine:

-It is vitamin B1

-It is naturally found in many foods in the human diet. In this case, the patient consumes excess alcohol- so he may get thiamine deficiency due to poor nutrition and lack of essential vitamins due to impaired ability of the body to absorb these vitamins.

-Indications: Given to this patient mainly to prevent Wernickes encephalopathy- that can lead to confusion, ataxia and opthalmoplegia.

7. Tab MVT:

-This is methylcobalamin. This patient requires the supplement due to the deficiency of Vit B12. 

3) Did the patients history of denovo hypertension contribute to his current condition?

-When there is blood flow restricted to the cerebellum, the main cause is most likely cerebellar infract. In cases of untreated hypertension, most commonly, which leads to high blood pressure, the formation of cerebellar infracts have a higher chance of incidence. The blood vessels are more prone to an external stress. Due to the endothelial dysfunction seen in this case, there can be an impaired response of the vessel. This are some of the etiological factors leading to stroke. Small vessel disease can be caused due to hypertension. 

 -Reference: https://www.ncbi.nlm.nih.gov/pmc/articles/PMC3741336/

4) Does the patients history of alcoholism make him more susceptible to ischaemic or haemorrhagic stroke?

- In the two types of stroke, ischemic and hemorrhagic, alcohol consumption can have a different effect on the risk factor. In ischemic stroke, which is a more common type of stroke, there is a clot which is formed which blocks the flow of blood and stops adequate oxygen from reaching the brain. In hemorrhagic stroke, there are leaks from the blood vessels or it could be due to a burst of aneurysms.

-According to a Cambridge study, heavy drinkers have 1.6 more chance of intracerebral hemorrhage and a 1.8 increased chance of subarachnoid hemorrhage. The adverse effect on BP that is seen due to increased drinking is a major stroke risk factor and increase the risk of heart stroke. Many studies show that with mild and moderate drinking. The risk of ischemic stroke decreases due to decreased level of fibrinogen which helps in the formation of blood clots. However, heavy alcohol intake is associated with impaired fibrinolysis, increased platelet activation and increased BP and heart rate. So In this case, his history of alcoholism, coupled with his hypertension definitely could be a causative factor of his current condition. 



Case #3 


1) What is the evolution of the symptomatology in this patient in terms of an event timeline and where is the anatomical localization for the problem and what is the primary etiology of the patient's problem?

10 years ago

Episode of right and left upper limb paralysis

1 year ago

Right and left paresis due to hypokalemia

8 months ago

Bilateral pedal edema, gradually progressing, present in both sitting and standing position, relieved on taking medication

7 months ago

Diagnosed with infection in the blood

2 months ago

Visited our hospital for neck pain and received medication

6 days ago

Pain in the left upper limb, radiating along the upper limb, dragging type, nocturnal increase in the pain, aggravated during palpitations and relieved on medication

 

5 days ago

Palpitations, sudden in onset, more during night time, aggravated by lifting weights and speaking continuously, relieved by drinking more water, medication

Dyspnea during palpitation.

Chest pain associated with chest heaviness.

 

 Anatomical location: 

-It is the cervical spine.

Primary Etiology:

-The patient experienced episodes of palpitations, paresis, paralysis and has edema because of hypokalemia. The neck pain is due to cervical spondylosis.

2)  What are the reasons for recurrence of hypokalemia in her? Important risk factors for her hypokalemia?

-In this patient, there is a presentation of hypoalbuminemia, which is a cause of edema and hypokalemia.

-Hypokalemia is a side effect seen in patients who are on diuretic medication, that can cause edema.

- Some of the risk factors are:

Diabetic Ketoacidosis

Diarrhea

Excessive Sweating

Excessive Laxative Use

Excessive Alcohol Use

Folic Acid Deficiency

Vomiting

Diuretics

CKD

Primary Aldosteronism


3) What are the changes seen in ECG in case of hypokalemia and associated symptoms?

- Some of the changes seen in the electrocardiogram is that there is a depression in the amplitude of the T-wave. ST segment depression and T wave inversions are noticed when the potassium levels further drop. The PR interval is elongated and the P wave amplitude is increased. The T wave is seen in mid-precordial leads, and there could be a positive deflection after the T-waves.


Case #4


1) Is there any relationship between occurrence of seizure to brain stroke. If yes what is the mechanism behind it?

-The most common cause of seizures in elderly patients is due to CVA or stroke. Stroke can be of two types:

    =Ischemic stroke: This happens due to a blood clot or a lack of blood flow to the brain.

    =Hemorrhagic stroke: This happens due to bleeding within or around the brain. People who’ve had a                     hemorrhagic stroke are more likely to have seizures after an episode of stroke than those who’ve had an              ischemic stroke. 

-Strokes which occur in the cerebral cortex or with a higher severity can lead to an increased incidence of stroke.

-Most seizure activity which follow an episode of stroke can again be of two types.

    =Early onset seizures: These can peak within 24 hours after stroke.

    =Late onset seizures: These tend to occur after 2 week of stroke onset, peak within 6-12 months after the              stroke. This is the type of stroke which has a higher incidence of onset. 

-After an ischemic stroke, when there are early onset seizures, these are due to an increase in intracellular Ca2+ and Na+ with a resultant lower threshold for depolarization. Glutamate excitotoxicity, hypoxia, metabolic dysfunction, destruction of phospholipid membrane, secretion of free fatty acids in penumbras areas. These may lead to disruption of electrical activity in the brain. 

-After hemorrhagic strokes, there could be an irritation due to the productions present in the blood, through blood metabolism, such as hemosiderin deposits. 

-Late onset seizures are can lead to changes in the neuronal excitability and glottic scarring.

 

2) In the previous episodes of seizures, patient didn't loose his consciousness but in the recent episode he lost his consciousness what might be the reason?

The patient has a history with seizure activity, so due to this there is an increase in the chemical and mechanical activity , which leads to the development of organic lesions in the brain. The bigger the lesions the more severe the symptoms.

-Since this patient has had recurrent seizures, there might be an aggravation on symptoms during this episode compared to the last. Loss of consciousness in this patient may be due to this. 


Case #5

https://nikhilasampathkumar.blogspot.com/2021/05/a-48-year-old-male-with-seizures-and.html?m=1


1) What could have been the reason for the patient for developing ataxia in the past 1 year?

-One reason for this patient to develop ataxia is the history of alcohol consumption in this patient. Due to the excessive consumption in this patient, there is a higher chance of developing cerebellar ataxia. 

-A potential mechanism for this is alteration in GABA-A receptor dependent neurotransmission. Ethanol is shown to disrupt molecular events at the mossy fibre-granule cell-golgi cell synaptic site and the granule cell fibre-Purkinje cell synaptic site, which is mainly responsible for ethanol induced cerebellar ataxia.

-Another mechanism is the relation between age related effect of ethanol on the endoplasmic reticulum of purkinje cells of dendrite causing dendritic regression, and the effect of ethanol withdrawal that causes mitochondrial damage in the cerebellum.

-Ethanol also causes neuroinflammation and neurotoxicity in the cerebellum.

-These can all affect the cerebellum, which is the motor coordination centre of the central nervous system, and also involved in cognitive processing and sensory discrimination. These can all result in altered hand movements, impaired postural stability and balance, loss of fine movements etc.

 

 

2) What was the reason for his IC bleed? Does alcoholism contribute to bleeding diathesis?

-The adverse effect on BP that is seen due to increased drinking is a major stroke risk factor and increase the risk of heart stroke. Heavy drinking is a major cause of the acute cerebral hemorrhage of frontal, parietal and temporal lobes in this patient.

-This patient has a history of excessive alcohol consumption for the past three years. According to a study, heavy drinkers have 1.6 more chance of intracerebral hemorrhage and a 1.8 increased chance of subarachnoid hemorrhage. 

-Bleeding diathesis is an unusual susceptibility to bleed which can be due to mainly due to hypercoagulability. Heavy drinking can cause thrombocytopenia, as well as impact shape and functions of platelets. Impaired platelet function, together with reduced platelet count, can contribute to this condition associated with chronic alcoholism. 


Case #6

http://shivanireddymedicalcasediscussion.blogspot.com/2021/05/a-30-yr-old-male-patient-with-weakness.html

1) Does the patient’s history of road traffic accident have any role in his present condition?

-Like the attached link reference, the patients history with a RTA could have influenced his left cerebral infract in the internal capsule.

-Reference: https://www.ahajournals.org/doi/pdf/10.1161/01.STR.14.4.617

2) What are warning signs of CVA?

-The warning signs of CVA are: 

Severe Headache

Trouble seeing out of one/both eyes

Trouble walking

Sudden numbness or weakness

Sudden confusion


3) What is the drug rationale in CVA?

i) Antiplatelet therapy: Due to the thrombotic origin and the involvement of platelet aggregation in the development of said thrombus, antiplatelet drugs play an important role in the medical treatment. The most widely used antiplatelet agent is NSAID acetylsalicylic acid.

Aspirin at low doses binds and inhibits the platelet COX-1 irreversibly and consequently impairs the production of prostaglandins and thromboxanes, noting thromboxane A2 in particular. The absence of TXA2 leads to the reduction in the TXA2-mediated amplification of platelet activation and thus stops the platelet aggregation that includes morphological changes of the fibrinogen receptor which is required for platelet aggregation.

ii) Thrombolysis:  Therapy with rtPA is given at a dose of 0.9 mg/kg IV without exceeding a maximum dose of 90 mg with 10% given as a loading bolus over 1 minute and the remainder as an infusion over 60 minutes. During the infusion and for one hour after concluding the infusion, the patient’s vital signs should be monitored and neurological assessment done every 15 minutes. Thereafter, observations should be carried out every 30 min for the next 6 hours and hourly afterward until 24 hours have transpired since treatment.

iii) Anticoagulant therapy: Anticoagulants are a group of drugs that by interacting with the coagulation cascade disrupt the formation of the fibrin mesh that forms the scaffold of the clot. When in homeostasis, the blood elements that participate in this process are kept at check thus preventing the formation of a blood clot in situ, or thrombus, inside the blood vessels.


4. Does alcohol has any role in his attack?

- https://www.ncbi.nlm.nih.gov/pmc/articles/PMC6007300/

- According to the above study, patients who consume 1-21 drinks a week, have a lower chance of developing ischemic or hemorrhagic stroke than those who are heavy drinkers. The patient is an occasional alcohol drinker, so the chances of alcohol affecting his attack is improbable. In heavy drinkers, alcohol can increase the chances of both types of strokes.

 

5.Does his lipid profile has any role for his attack?

- The patient has an overall normal lipid profile except for the HDL count. The HDL is 33mg/dl which is lower than the normal range (40-60 mg/dl).

-HDL is known as the good cholesterol. Any decrease in the count is an indicator that there can be a cardiovascular disorder.

-Studies have demonstrated a trend toward a higher risk of stroke with lower HDL-C. Some see HDL-C as an important modifiable stroke risk factor. In patients with recent stroke or transient ischemic attack and no coronary heart disease, only lower baseline HDL-C predicted the risk of recurrent stroke.

https://pubmed.ncbi.nlm.nih.gov/21830454/#:~:text=Studies%20have%20demonstrated%20a%20trend,the%20risk%20of%20recurrent%20stroke


Case #7

https://amishajaiswal03eloggm.blogspot.com/2021/05/a-50-year-old-patient-with-cervical.html 

 

1) What is myelopathy hand?

-There is loss of power of adduction and extension of the ulnar two or three fingers and an inability to grip and release rapidly with these fingers. These changes have been termed "myelopathy hand" and appear to be due to pyramidal tract involvement.

-A characteristic dysfunction of the hand has been observed in various cervical spinal disorders .

 


2) What is finger escape?

-It is one of the signs in cervical cord damage, seen in Cervical MyelopathyWhen patient holds fingers extended and adducted, the small finger spontaneously abducts due to weakness of intrinsic muscle.

- This commonly results from weakness of the ulnar nerve innervated intrinsic hand muscles particularly Palmar Interosseous Muscle. It is a component of Wartenberg’s sign-consisting of involuntary abduction of the fifth finger, caused by unopposed action of the extensor digiti minimi.

  



 

3.What is Hoffman’s reflex?

-Hoffman's sign is also called snapping reflex, digital reflex, Jacobson's reflex. It is used to examine the reflexes of upper extremities.

-The doctor does the test by:

-Result: If there is no movement in the index finger or thumb after this motion, the person has a negative Hoffman’s sign. If the index finger and thumb move, the person has a positive Hoffman’s sign.

-A positive Hoffman sign indicates an upper motor neuron lesion and corticospinal pathway dysfunction likely due to cervical cord compression. (Eg: Cervical myelopathy). A positive Hoffman sign can be present in a normal patient. This is seen in individuals with hyperreflexia. 

Case #8

1  1) What can be the cause of her condition?  

-The patient’s GTCS episodes can be due to acute cortical vein thrombosis as seen in her MRI.

-A very common symptom of CVT is seizure activity. This case report shows that CVT can occur when there is thrombocytopenia and anemia. Headache and vomiting are common symptoms seen in midline shift cases. 

-According to the linked case, the neurological symptoms doesn't have to corelate to the Iron deficiency anemia. With the presentation of thrombocytopenia, the common diagnosis would be an increased bleeding tendency in the patient, but instead paradoxically it has lead to venous thrombosis. This can be elucidated in the above case.

-Reference: https://www.ncbi.nlm.nih.gov/pmc/articles/PMC5771304/

2.    2) What are the risk factors for cortical vein thrombosis?

Tumor

Dehydration

Protein Deficiencies

Excess Estrogen

Infection

Obesity

Cancer

3.  3) There was seizure free period in between but again sudden episode of GTCS why? Resolved spontaneously why?      

-This patient has a history of thrombophlebitis, which leads to a high grade of fever. This can be the root of the development of seizures. Resolution of the seizures, can occur when there is a dip in the fever.

4. 4) What drug was used in suspicion of cortical venous sinus thrombosis?

-This patients treatment consists mainly of 3 treatment choices:

        = Thrombolysis 

        = Anticoagulation (Intravenous/ Subcutaneous LMW Heparin)

        = Symptomatic Treatment

-Any of the above treatment options can help resolve cortical venous sinus thrombosis. 




3) CARDIOLOGY:

Case #1:

https://muskaangoyal.blogspot.com/2021/05/a-78year-old-male-with-shortness-of.html.

1) What is the difference btw heart failure with preserved ejection fraction and with reduced ejection fraction?

-Ejection fraction is defined as a measurement, which is expressed as a percentage. It shows how much blood the left ventricle pumps out with each contraction of the heart. Heart Failure with preserved ejection fraction is termed as HFpEF, this is also know as diastolic HF. Whereas when the Heart Failure is associated with reduced ejection fraction, it is termed as HFrEF, this is also known as systolic HF. Fatigue and shortness of breath are common symptoms of both HFpEF and HFrEF.

-In HFpEF, there is normal contraction of the heart and an adequate amount of blood is pumped from the heart, but there is hypertrophy of the cardiac musculature in the ventricle. This will lead to a small volume of blood being able to be held in the chamber. The heart cannot meet the body's requirement even if the output of blood appears to be the same. Coronary hypertrophy as seen in this case is seen in CAD, Aortic Stenosis, and Hypertrophic Cardiomyopathy.

-In HFrEF, there is an inadequate contraction of the muscle due to the dilatation of the chamber. This leads to the release of less oxygenated blood. Patients with this type of heart failure will have lower-than-normal left ventricular ejection fraction on an echocardiogram. Coronary hypertrophy as seen in these cases are Valvular heart disease, diabetes, and hypertension.

-For reference values: LVEF of 50% to 75% indicates a normal pumping ability; 36% to 49% is considered below normal; 35% or lower is considered low pumping ability.                                                 


2) Why haven't we done pericardiocentesis in this patient?  

-Pericardiocentesis is a procedure done to remove fluid that has built up in the sac around the heart . It's done using a needle and small catheter to drain excess fluid.

-There are 3 mainly used approaches for needle entry - left parasternal, left apical approach and subxyphoid approach. 

-These steps should be performed with caution, to avoid injury to the pleura, liver or diaphragm. 


-In this patient, symptomatic treatment was preferred when compared to invasive procedures because the pericardial effusion can be graded as mild to moderate. 
 


3) What are the risk factors for development of heart failure in the patient?

I) SMOKING: The patient in the above case is a known chronic smoker for the past 30 years. A known risk factor for heart failure is smoking.

-MOA: Cigarette smoking leads to impaired endothelial function via decreased nitric oxide production, pro-thrombotic state, increased oxidative stress, and activated inflammatory pathways. Along with the vascular effect, smoking, through increased oxidative stress and inflammation, effects on the myocardium leading to systolic and diastolic dysfunction. It also promotes other heart failure risk factors including blood pressure, increased heart rate, diabetes, and atherosclerosis. 

II) DM and HTN

-Diabetes results in drastic changes in myocardial structure and function.

-It can lead to main interstitial/perivascular fibrosis and left ventricular hypertrophy.These changes thicken the heart's musculature, and that will result in diastolic and systolic dysfunction and increase risk of heart failure.

-Hypertension increases work load on the heart and a result there is left ventricular hypertrophy. This in turn increases the risk of heart failure. 

III) ALCOHOL 

-Heavy alcohol consumption is known to be associated with alcoholic cardiomyopathy 

- Alcoholic cardiomyopathy is characterized by left ventricular dilation, reduced or normal left ventricular wall thickness among patients and increased left ventricular mass. 

IV) 2D ECHO RESULTS 

-The results of the 2D echo of the patient shows pericardial effusion. This increases pressure on the heart. Without treatment if will end up as heart failure. 

V) ECG REPORTS 

-The reports of the patient show first degree AV block. This is associated with an increased risk of heart failure.

-Among patients with heart failure, first-degree atrioventricular block is present in anywhere between 15% and 51%. 




4) What could be the cause for hypotension in this patient?

-Hypotension in this patient could be due to combination of the presence of pericardial effusion and use of diuretic LASIX. This patient is on medication for hypertension.

Case #2

https://muskaangoyal.blogspot.com/2021/05/a-73-year-old-male-patient-with-pedal.html

1) What are the possible causes for heart failure in this patient?

The patient has multiple comorbidities. These could have led to a heart failure. 

1. The patient was diagnosed with type 2 diabetes mellitus 30 years ago and has been taking insulin daily. The patient was also diagnosed with diabetic triopathy. This shows that there was uncontrolled diabetes which is major risk factor for heart failure.

-Reference:  https://www.ncbi.nlm.nih.gov/pmc/articles/PMC5494155/

2. The patient was also diagnosed with hypertension 19 years ago. This is another risk factor for heart failure.

-Reference:https://pubmed.ncbi.nlm.nih.gov/31472888/

3. He is a chronic alcoholic since 40 years which is a risk factor towards heart failure

-Reference: https://www.nmcd-journal.com/article/S0939-4753(19)30360-6/fulltext

- According to the article there is evidence that moderate and heavy alcohol consumption are associated with decreased LVEF and trend towards a higher risk of incident LV systolic dysfunction, compared to lighter drinkers.

4. The patient has elevated creatinine and AST/ALT ratios is more than 2 and was diagnosed with chronic kidney disease stage IV. CKD is another risk factors for heart failure.

-Reference: https://www.ncbi.nlm.nih.gov/pmc/articles/PMC2900793/

 

2) What is the reason for anemia in this case?

-The patient has normocytic normochromic anemia. It could be anemia of a chronic disease as the patient is diagnosed with CKD stage IV. Chronic kidney disease results in decreased production of erythropoietin which in turn decreases the production of red blood cells from the bone marrow.



-Patient’s with anemia and CKD also tend to have deficiency in nutrients like iron, vitamin B12 and folic acid essential in making healthy red blood cells


 

3) What is the reason for blebs and non-healing ulcer in the legs of this patient?

-The most common cause for blebs and non-healing ulcer in this patient is diabetes mellitus. 

-CKD is also known to cause delay in healing of wounds along with poorly controlled diabetes. 

-Anemia present in this patient can also delay the healing of wounds.

                                                                                                                

4) What sequence of stages of diabetes has been noted in this patient?

-There are mainly 4 stages in type 2 diabetes- 


- The patient is diagnosed with diabetic triopathy, showing signs of diabetic neuropathy, retinopathy and nephropathy. 

Case #3

https://preityarlagadda.blogspot.com/2021/05/biatrial-thrombus-in-52yr-old-male.html

1) What is the evolution of the symptomology in this patient in terms of an event timeline and where is the anatomical localization for the problem and what is the primary etiology of the patients problem?

Timeline of the patient is as follows:

1 year ago

 SOB (Grade 2) Diagnosed with HTN

2 days ago

SOB (Grade 2 progressed to Grade 4) ; Decreased urine output

Same day

SOB (Grade 4); Anuria (for 1 day)


Anatomical Location:

-Patient has an problem that can be localized as an issue in the cardiac musculature

Etiology:

-Congestive heart failure is a chronic progressive condition that affects the pumping ability of the cardiac muscle. It occurs if the heart cannot pump or fill adequately. Loss of atrial contraction and left atrial dilation in this case cause stasis of blood in the left atrium and may lead to thrombus formation in the left atrial appendage. 


2) What are the mechanism of action, indication and efficacy over placebo of each of the pharmacological and non pharmacological interventions used for this patient?

1. TAB. Cardivas 3.125mg PO/BD
MOA- It is Carvediol. Carvedilol inhibits exercise induce tachycardia through its inhibition of beta adrenoceptors. Carvedilol's action on alpha-1 adrenergic receptors relaxes smooth muscle in vasculature, leading to reduced peripheral vascular resistance and an overall reduction in blood pressure. At higher doses, calcium channel blocking and antioxidant activity can also be seen. The antioxidant activity of carvedilol prevents oxidation of low density lipoprotein and its uptake into coronary circulation.

Indications- Used as a long term drug to reduce mortality in patients with congestive heart failure.

2. TAB. Dytor 10mg PO/OD
-MOA- It is torsemide. Torasemide is part of the loop diuretics and thus, it acts by reducing the oxygen demand in the medullary thick ascending loop of Henle by inhibiting the Na+/K+/Cl- pump on the luminal cell membrane surface. This action is obtained by the binding of torasemide to a chloride ion-binding site of the transport molecule.

Torasemide is known to have an effect in the renin-angiotensin-aldosterone system by inhibiting the downstream cascade after the activation of angiotensin II. This inhibition will produce a secondary effect marked by the reduction of the expression of aldosterone synthase, TGF-B1 and thromboxane A2 and a reduction on the aldosterone receptor binding. 

-Indications- preferred in cases of hypertension associated with CCF and renal failure.


3. TAB Pan D 40mg PO/OD
-MOA- It is a combination of domperidone and pantaprazol. It is a proton pump inhibitor and helps decrease acid production in the stomach.
-Indications- It is used to treat gastroesophageal reflux disease (Acid reflux) and peptic ulcer disease by relieving the symptoms of acidity such as indigestion, heartburn, stomach pain, or irritation.

4. TAB. Taxim 200mg PO/OD
-MOA- It is cefixime. Like all beta-lactam antibiotics, cefixime binds to specific penicillin-binding proteins located inside the bacterial cell wall, causing the inhibition of the third and last stage of bacterial cell wall synthesis. Cell lysis is then mediated by bacterial cell wall autolytic enzymes such as autolysins; it is possible that cefixime interferes with an autolysin inhibitor.
-Indications- Given mainly to prevent development of bacterial infections.

5. INJ. Thiamine 100mg in 50ml NS IV/TID
-It is vitamin B1. It is naturally found in many foods in the human diet. In this case, the patient consumes excess alcohol- so he may get thiamine deficiency due to poor nutrition and lack of essential vitamins due to impaired ability of the body to absorb these vitamins.

6. INJ. HAI S.C 8U-8U-6U
Insulin given in this case to treat the patients denovo diabetes mellitus. 

7. INJ. Dobutamine- 
-MOA- It is a synthetic catecholamine, that acts on B1, B2 and alpha 1 receptors.
-Indications- Indicated when parenteral therapy is necessary for inotropic support in the short-term treatment of patients with cardiac decompensation due to depressed contractility resulting either from organic heart disease or from cardiac surgical procedures.

8. TAB. Digoxin-
- MOA- Digoxin exerts hemodynamic, electrophysiologic, and neurohormonal effects on the cardiovascular system. It reversibly inhibits the Na-K ATPase enzyme, leading to various beneficial effects. The Na-K ATPase enzyme functions to maintain the intracellular environment by regulating the entry and exit of sodium, potassium, and calcium (indirectly). Na-K ATPase is also known as the sodium pump. The inhibition of the sodium pump by digoxin increases intracellular sodium and increases the calcium level in the myocardial cells, causing an increased contractile force of the heart. This improves the left ventricular ejection fraction (EF), an important measure of cardiac function.
Indications- Digitalis is used in patients with low output failure especially when associated with atrial fibrillation, as indicated in this case.
- Indications: Digoxin is indicated in the following conditions: 
                    1) For the treatment of mild to moderate heart failure in adult patients. 
                    2) To increase myocardial contraction in children diagnosed with heart failure. 
                    3) To maintain control ventricular rate in adult patients diagnosed with chronic atrial fibrillation.

9. INJ. Unfractionated Heparin 5000-
-MOA- At low concentration, heparin selectively inhibits the conversion of prothrombin to thrombin, thus preventing thrombus formation. High dose heparin has antiplatelet action and prolongs bleeding time.
Indications- Patient had a biatrial thrombus and in this case it was used to prevent further thrombus formation.

10. TAB. Carvediol 3.125mg BD
-MOA- It blocks B1, B2, Alpha 1 adrenergic receptors and no intrinsic sympathomimetic activity.
Indications- Used as a long term drug to reduce mortality in patients with congestive heart failure.

11. TAB. Acetyl cysteine 600mg PO TID
- MOA- A number of possible mechanisms for the mucolytic activity of acetylcysteine have been proposed. Acetylcysteine's sulfhydryl groups may hydrolize disulfide bonds within mucin, breaking down the oligomers, and making the mucin less viscous. Acetylcysteine has also been shown to reduce mucin secretion in rat models. It is an antioxidant in its own right but is also deacetylated to cysteine, which participates in the synthesis of the antioxidant glutathione. The antioxidant activity may also alter intracellular redox reactions, decreasing phosphorylation of EGFR and MAPK, which decrease transcription of the gene MUC5AC which produces mucin.

12.  TAB. Acitrom 2mg OD 
-MOA- It is an anticoagulant that functions as a vitamin K antagonist.
Indications- oral anticoagulant which helps to prevent formation of harmful blood clots in the legs, lungs, brain and heart. It is used for deep vein thrombosis, pulmonary embolism and stroke prevention.


3) What is the pathogenesis of renal involvement due to heart failure (cardio renal syndrome)? Which type of cardio renal syndrome is this patient?

-Cardiorenal syndrome is basically defined as any acute or chronic problem in the heart or kidneys that could result in an acute or chronic problem of the other.

-The leading cause of CHF includes ischemic heart diseases and myocardial infarction, diabetes mellitus, the metabolic syndrome and hypertension. 

-CHF evolves due to a single cause, such as myocardial infarction or a cumulative process of multiple minor effects. Often one entity is poorly controlled and causes significant system stress. There is immediate stress on the kidney through pathophysiological connections when CHF develops. The connectivity of the vascular bed, and its regulation by the sympathetic nervous system  and RAAS, continues the stress on the nephron. Fibrosis can occur in the organs long term. 

-An important source of renal stress is increased cardiac preload. The kidneys receive 25% of blood flow, where the majority goes to the cortex, which also has the greatest neural innervations to regulate changes acutely. The medulla receives only 10% of the blood supply. The renal microvascular bed however is continuous throughout. Disease in any glomeruli could have implications when placed under stress from SNS or RAAS and matched with early disease in vascular endothelium.

-Compensation to ensure adequate GFR includes increased renal blood flow (afferent arteriolar vasodilatation), filtration pressure (via efferent arteriolar vasoconstriction) and glomerular hypertrophy, and hyperfiltration (leads to scarring).

In this case the patient has Type 4 cardiorenal syndrome: a chronic decline in kidney function that results in chronic cardiac dysfunction.

4) What are the risk factors for atherosclerosis in this patient?

- In this case, the risk factors for the development of atherosclerosis include:

i. Patient has Diabetes mellitus type 2, which can accelerate atherosclerosis by driving inflammation and slowing down blood flow.

ii.  Patient has history of alcohol abuse that can lead to atherosclerosis and increase the risk of stroke.

iii.  Patient has a history of NSAID abuse, which can change the vessels ability to relax and also stimulate growth of smooth muscle cells inside the arteries, thus leading to the clogging of the arteries.

iv. Patient also has a history of hypertension- effect on the arterial wall also results in the aggravation and acceleration of atherosclerosis, particularly of the coronary and cerebral vessels. Moreover, hypertension appears to increase the susceptibility of the small and large arteries to atherosclerosis.

5) Why was the patient asked to get those APTT, INR tests for review?

-APTT: Activated partial thromboplastin time. This is a blood test that characterizes coagulation of blood. The patient has a propensity for thrombus formation, which needs to be monitored by keeping check on the aPTT levels which is an indicator for the coagulability of the blood.

-INR: It is international normalized ratio; it is also a measure of the ability of the blood to clot. This is an important test for patients who are on blood thinners anticoagulants. The patient in this case was taking heparin, so everyday reports of his INR value were needed.



 

Case #4


1) What is the evolution of the symptomatology in this patient in terms of an event timeline and where is the anatomical localization for the problem and what is the primary etiology of the patient's problem?

The timeline of events in this patient is:

12 years ago

DM (on medication)

1 year ago

Heart burn (relieved without medication)

7 months ago

Pulmonary TB

6 months ago

HTN

Half an hour

SOB (even at rest)


Anatomical localization: 
- This effects the Cardiovascular system.

Etiology:  
-The patient is hypertensive and diabetic. Both these conditions can cause atherosclerosis. Atherosclerosis is the build up of fatty and fibrous material inside the wall of arteries.

2) What are mechanism of action, indication and efficacy over placebo of each of the pharmacological and non pharmacological interventions used for this patient?

Pharmacological interventions:

-TAB MET XL 25 MG/STAT-contains Metoprolol as active ingredient

-MOA: METOPROLOL is a cardioselective beta blocker

-Beta blockers work by blocking the effects of the hormone epinephrine, also known as adrenaline. Beta blockers cause your heart to beat more slowly and with less force. Beta blockers also help open up your veins and arteries to improve blood flow.

-Indications: it is used to treat Angina, High blood pressure and to lower the risk of hear attacks .

-Efficacy based on studies are:  Patients were randomized to one of four treatment arms: placebo or ER metoprolol (0.2 mg/kg, 1.0 mg/kg, or 2.0 mg/kg). Data were analyzed on 140 intent-to-treat patients. Results:  mean baseline BP was 132/78 +/- 9/9 mmHg. Following 4 weeks of treatment, mean changes in sitting BP were: placebo = -1.9/-2.1 mmHg; ER metoprolol 0.2 mg/kg = -5.2/-3.1 mmHg; 1.0 mg/kg = -7.7/-4.9 mmHg; 2.0 mg/kg = -6.3/-7.5 mmHg. Compared with placebo, ER metoprolol significantly reduced systolic blood pressure (SBP) at the 1.0 and 2.0 mg/kg dose (P = .027 and P = .049, respectively), reduced diastolic blood pressure (DBP) at the 2.0 mg/kg dose (P = .017), and showed a statistically significant dose response relationship for the placebo-corrected change in DBP from baseline. There were no serious adverse events or adverse events requiring study drug discontinuation among patients receiving active therapy.

 -Non pharmacological intervention advised to this patient is: Percutaneous Coronary Intervention.

3) What are the indications and contraindications for PCI?

   - INDICATIONS:

    



    - Contraindications: 



4) What happens if a PCI is performed in a patient who does not need it? What are the harms of overtreatment and why is research on overtesting and overtreatment important to current healthcare systems?

Although PCI is generally a safe procedure , it might cause serious certain complications like



-Due to of all these complications it is better to avoid PCI in patients who do not require it.

-Research on over-testing and over-treatment is important as they are more harmful than useful.

Harms to patients

-Performing screening tests in patients with who at low risk for the disease which is being screened.

For example:Breast Cancer Screenings Can Cause More Harm Than Good in Women Who Are at Low Risk. A harmless lump or bump could incorrectly come up as cancer during routine breast screenings. This means that some women undergo surgery, chemotherapy or radiation for cancer that was never there in the first place.

-Overuse of imaging techniques such as X- RAYS AND CT SCANS as a part of routine investigations.

-Overuse of imaging can lead to a diagnosis of a condition that would have otherwise remained irrelevant - OVER-DIAGNOSIS.

Also the adverse effects due to this are more when compared to the benefits.

-Over-diagnosis through overtesting can psychologically harm the patient.

-Hospitalizations for those with chronic conditions who could be treated as outpatients[ can lead to economic burden and a feeling of isolation.

Harm to health care systems

-The use of expensive technologies and machineries are causing economic burden on health care systems.


Case #5

https://bhavaniv.blogspot.com/2021/05/case-discussion-on-myocardial-infarction.html?m=1

1) What is the evolution of the symptomatology in this patient in terms of an event timeline and where is the anatomical localization for the problem and what is the primary etiology of the patient's problem?

 

3 days ago

Developed chest pain on the right side of the chest.

 

Anatomical Localization:

àCardiovascular system – Occlusion of the right coronary artery.

Etiology:

·         Atherosclerosis – Also known as coronary artery disease, this condition is the most common cause of heart attacks and occurs when the buildup of fat, cholesterol, and other substances forms plaque on the walls of the coronary arteries

·         Coronary artery spasm – A rare cause of blockage, spasms of the coronary arteries can cause them to become temporarily constricted. 

·         Coronary artery tear – Also known as a spontaneous coronary artery dissection, a tear in a coronary artery can prevent blood from reaching the heart and cause a heart attack.

 

2) What are mechanism of action, indication and efficacy over placebo of each of the pharmacological and non pharmacological interventions used for this patient?

·         1.TAB. ASPIRIN 325 mg PO/STAT

-MOA- Aspirin is a NSAID. They inhibit COX-1 and COX-2 thus decreasing the prostaglandin level and thromboxane synthesis.

-Indications- They are anti platelet medications and, in this case, used to prevent formation of blood clots in blood vessels.

-Efficacy over Placebo: According to the study, there was a clear reduction in some serious cardiovascular adverse events.  Aspirin use was associated with a lower risk of myocardial infarction than placebo use or no treatment (risk ratio [RR], 0.83, 95% confidence interval [CI]: 0.73–0.95, P = 0.005).

-Reference: https://www.ncbi.nlm.nih.gov/pmc/articles/PMC6767763/  

·         2.TAB ATORVAS 80mg PO/STAT

-Indication: Atorvastatin is indicated for the treatment of several types of dyslipidemias. Dyslipidemia describes an elevation of plasma cholesterol, triglycerides or both as well as to the presence of low levels of high-density lipoprotein. This condition represents an increased risk for the development of atherosclerosis. Atorvastatin is indicated, in combination with dietary modifications, to prevent cardiovascular events in patients with cardiac risk factors and/or abnormal lipid profiles. Atorvastatin can be used as a preventive agent for myocardial infarction, stroke, and angina, in patients without coronary heart disease but with multiple risk factors and in patients with type 2 diabetes without coronary heart disease but multiple risk factors. Atorvastatin may be used as a preventive agent for non-fatal myocardial infarction, fatal and non-fatal stroke, revascularization procedures, hospitalization for congestive heart failure and angina in patients with coronary heart disease.

-MOA: Atorvastatin is a statin medication and a competitive inhibitor of the enzyme HMG-CoA reductase, which catalyzes the conversion of HMG-CoA to mevalonate, an early rate-limiting step in cholesterol biosynthesis. Atorvastatin acts primarily in the liver, where decreased hepatic cholesterol concentrations stimulate the upregulation of hepatic low-density lipoprotein receptors, which increases hepatic uptake of LDL. Atorvastatin also reduces Very-Low-Density Lipoprotein-Cholesterol, serum triglycerides and Intermediate Density, but increases High-Density Lipoprotein Cholesterol. In vitro and in vivo animal studies also demonstrate that atorvastatin exerts protective effects independent of its lipid-lowering properties, also known as the pleiotropic effects of statins. These effects include improvement in endothelial function, enhanced stability of atherosclerotic plaques, reduced oxidative stress and inflammation, and inhibition of the thrombogenic response. Statins were also found to bind allosterically to β2 integrin function-associated antigen-1, which plays an essential role in leukocyte movement and T cell activation.

-Efficacy over Placebo: Out of 18 studies done, statins were shown to help in 16 studies. The studies show a 27% reduction in the onset of MI. https://www.ncbi.nlm.nih.gov/pmc/articles/PMC5895306/

       3. TAB CLOPIBB 300mg PO/STAT

-MOA- Clopidogrel is metabolized to its active form by carboxylesterase-1.3 The active form is a platelet inhibitor that irreversibly binds to P2Y12 ADP receptors on platelets. This binding prevents ADP binding to P2Y12 receptors, activation of the glycoprotein GPIIb/IIIa complex, and platelet aggregation.

-Indications- Clopidogrel is indicated to reduce the risk of myocardial infarction for patients with non-ST elevated acute coronary syndrome, patients with ST-elevated myocardial infarction, and in recent MI, stroke, or established peripheral arterial disease.   


3) Did the secondary PTCA do any good to the patient or was it unnecessary?

PTCA is known to improve the patient’s vessel patency if it is done within 4 hours of the symptom onset or if it is used as adjunctive therapy along with some systemic thrombolytic therapy. It can restore up to 90% of the vessel’s natural state if implemented within enough time.

- Though there are certain benefits from PTCA, there are some disadvantages too. If done along with systemic thrombolytics then it can lead to a higher incidence of bleeding complications. Just PTCA alone, has not proven to show any ventricular function improvement or decreased mortality.

https://pubmed.ncbi.nlm.nih.gov/2971441/

Case #6

https://kattekolasathwik.blogspot.com/2021/05/a-case-of-cardiogenic-shock.h

1. How did the patient get relieved from his shortness of breath after i.v fluids administration by rural medical practitioner?

- Rapid breathing is an indicator of cardiogenic shock, if the patient also presents along with other signs such as cold, clammy extremities.

- In cardiogenic shock, there is hypovolemia, this will reduce perfusion to major organs in the body, when there is decreased perfusion, the body slows starts shutting down. To halt this process, iv fluids are given rapidly to continue the perfusion of fluids at the normal rate.

- When this patient was given fluids, the perfusion returns to normal which helps abate the shortness of breath.

 

2. What is the rationale of using torsemide in this patient?

- In patients who have cardiorenal syndrome, there is a renal dysfunction along with cardiac abnormalities. In such patients there is a volume overload and heart failure, the combination of increased pulmonary artery or central venous pressure with low systemic pressure may lead to a severe compromise of the net renal perfusion pressure.

- Furosemide is a commonly used diuretic to treat volume overload state in heart failure, yet it is particularly prone to the problem of diuretic resistance because of its particular pharmacokinetics. 

- Unfortunately, chronic diuretic use also induces hypertrophy in distal tubular cells, leading again to enhanced sodium reuptake, contributing further to diuretic resistance. Alternatives to furosemide, such as torsemide, have been shown to have a slight advantage in selected studies because of somewhat more favourable pharmacokinetics.

 

https://www.ncbi.nlm.nih.gov/pmc/articles/PMC2794438/

 

3. Was the rationale for administration of ceftriaxone? Was it prophylactic or for the treatment of UTI?

Patients with cardiorenal syndrome are known to have systemic inflammation which can be drawn parallel to end stage kidney disease. Here there is an inflammation of monocytes and other inflammatory cells. This puts the patient in a immune suppressive state.

- Due to this state, do reduce the chances of infection, as a prophylactic measure, ceftriaxone might have been started.

-Reference: https://www.ncbi.nlm.nih.gov/pmc/articles/PMC6165938/

 

4) GASTROENTEROLOGY:

Case #1:

https://63konakanchihyndavi.blogspot.com/2021/05/case-discussion-on-pancreatitis-with.html

1) What is the evolution of the symptomatology in this patient in terms of an event timeline and where is the anatomical localization for the problem and what is the primary etiology of the patient's problem?

The timeline of the events are:

5 years ago

Pain abdomen and Vomiting

3 years

Symptom free

1 year ago

5-6 episodes pain abdomen and vomiting

1 week ago

Alcohol binge

Since 1 week

 Pain Abdomen and vomiting

4 days since

High Grade fever with chills and rigors

Constipation

Burning micturition


Anatomical localization:

-This is mainly affecting the GIT, which specifically includes the stomach and pancreas. 

 Etiology:  

-The patient is a chronic alcoholic, episodes of abdominal pain and vomiting are following alcohol consumption. It can be assumed that it is heavy drinking that has led to the above condition.

 

2) What is the efficacy of drugs used along with other non pharmacological  treatment modalities and how would  you approach this patient as a treating physician?

Drugs used in this patient -

1. ING. MEROPENEM 

- Indication: Meropenem is a broad spectrum carbipenem antibiotic used to treat abdominal and skin infections.

-MOA: The bactericidal activity of meropenem results from the inhibition of cell wall synthesis. Meropenem readily penetrates the cell wall of most Gram-positive and Gram-negative bacteria to reach penicillin-binding- protein (PBP) targets. Its strongest affinities are toward PBPs 2, 3 and 4 of Escherichia coli and Pseudomonas aeruginosa; and PBPs 1, 2 and 4 of Staphylococcus aureus.

BASED ON A STUDY-In patients with moderate to severe intra-abdominal infections, empirical monotherapy with meropenem achieved clinical response rates ranging from 91 to 100% in 7 randomized comparative trials. Meropenem also achieved clinical response rates of over 80% in patients with severe intra-abdominal infections.

 

2. ING. METROGYL 500 mg IV TID for 5 days

- It contains Metronidazole. 

-Indication: Metronidazole belongs to Nitroimidazole group of antibiotics, is used to treat gastrointestinal infections, skin and blood infections.

-MOA: The exact mechanism of action of metronidazole has not been fully established, however, it is possible that an intermediate in the reduction of metronidazole which is only made by anaerobic bacteria and protozoa, binds deoxyribonucleic acid and electron-transport proteins of organisms, blocking nucleic acid synthesis. After administration, metronidazole enters cells by passive diffusion. Following this, ferredoxin or flavodoxin reduce its nitro group to nitro radicals. The redox potential of the electron transport portions of anaerobic or microaerophilic microorganisms renders metronidazole selective to these organisms, which cause nitro group reduction, leading to the production of toxic metabolites. These include N-(2-hydroxyethyl) oxamic acid and acetamide, which may damage DNA of replicating organisms.

-Based on the study below, metronidazole when combined with another antimicrobial agent is more effective in the treatment of complicated intra abdominal infections.

-Reference: https://www.ncbi.nlm.nih.gov

 3. ING. AMIKACIN 500 mg IV BD for 5days

-Amikacin is an amino glycoside antibiotic used in the treatment of serious bacterial infections.

 -All the above three antibiotics are given to control infection and prevent sepsis in the patient.

 

4. ING. OCTREOTIDE 100 mg SC , BD

-Octreotide is a long acting analogue of Somatostatin

-It inhibits exocrine secretion of the pancreas, also has anti inflammatory and cytoprotective effects.

-Indication: Octreotide by injection is used for the treatment of acromegaly and the reduction of flushing and diarrhea symptoms related to carcinoid tumors and/or vasoactive intestinal peptide tumors. The delayed-release oral formulation is used for the long-term treatment of acromegaly in patients who tolerate and respond adequately to injectable octreotide and lanreotide. 

-MOA: Octreotide's suppression of luteinizing hormone, reduction in splanchnic blood flow, and inhibition of serotonin, gastrin, vasoactive intestinal peptide, secretin, motilin, and pancreatic polypeptide provide relief for the gastrointestinal and flushing symptoms of carcinoid and/or VIPoma tumors. 

-Efficacy based on trials: Octeotride based on several studies did not provide any symptomatic relief or better cure when compared to other drugs . However it played a significant role in reducing serum lipase and amylase.

5. ING. PANTOP 

-Pantoprazole a proton pump inhibitor, is known to have pancreatic anti-secretory effect.

-Oxidative stress is common in acute pancreatitis- Pantoprazole  has a inhibitory effect on hydroxy radicals thereby reduces the progression of the disease and helps in reducing oxidative stress.

-PPZ treatment also reduces tissue infiltration of inflammatory cells and acinar cell necrosis in severe AP.

-Indication: Treatment of gastroesophageal reflux disease associated with a history of erosive esophagitis, Pathological Hypersecretion Associated with Zollinger-Ellison Syndrome, and Short-Term Treatment of erosive esophagitis associated with gastroesophageal reflux disease

 

6. ING. TRAMADOL in 100 ml NS  IV , OD

-Tramadol is an opioid analgesic used to relieve severe pain in acute pancreatitis.

 

7. ING. THIAMINE 100 mg in 100 ml NS  IV , TID*

-Vitamin B1 supplement. As the patient is on TPN there is a chance of B1 deficiency

- In Wernicke’s encephalopathy there has been several cases of pancreatitis. To prevent this Thiamine is given as a prophylactic measure.

 

8. TPN ( Total Parenteral Nutrition )

- TPN is a method of feeding that bypasses the gastrointestinal tract. Fluids are given intravenously to provide  nutrients the body needs. The method is used when a person cannot or should not receive feedings or fluids by mouth.

-Parenteral nutrition is used to prevent malnutrition in patients who are unable to obtain adequate nutrients by oral or enteral routes.

 9. IV NS / RL at the rate 12l ml per hour

 

Physician's Approach: 

-When the patients present with the complaints of pain abdomen and vomiting, along with fever, burning micturition, certain investigations must be done.

    --> First, a general examination must be done, including inspection, percussion, palpation and auscultation of the abdomen. Other investigations are CBP, LFT, RFT, Urine analysis, Serum amylase, ABG, and Pleural tapping. Some imaging studies like, contrast enhanced CT and chest x-ray should be taken as well.

    --> Now depending on the diagnosis based on the results, chemotherapy must be started. In the case of pancreatitis in this patient, the following treatment can be given. Antibiotics like Metrogyl, Amikacin and Meropenam. Fluid levels should be maintained with RL or NS. Somatostatin analogue like Octreocide, decreases the exocrine secretion in the pancreas. Proton pump inhibitor, vitamins such as Thiamine and anti-analgesic such as tramadol should also be given. 


Case #2:

https://nehae-logs.blogspot.com/2021/05/case-discussion-on-25-year-old-male.html

1) What is causing the patient's dyspnea? How is it related to pancreatitis?

- The dyspnea, or shortness of breath can be caused due do pancreatitis. 

-The release of inflammatory factors in the lungs, which occur due to the acute pancreatitis, will in turn lead to fluid accumulation. As seen as pleural effusion in this patient. 

-https://www.mayoclinic.org/diseases-conditions/pancreatitis/symptoms-causes/syc-20360227#:~:text=Breathing%20problems.,fall%20to%20dangerously%20low%20levels

2) Name possible reasons why the patient has developed a state of hyperglycemia.

i) Patient is an alcoholic with increased consumption since 2 months which could also be a cause of diabetes in the patient. But the patient was never tested before he came to our OPD and did not recall any notable signs.

ii) Pancreatitis damages cells that produce insulin and glucagon which are hormones that control the levels of blood sugar. Insufficiency of these hormones can lead to hyperglycemia. 

 

3) What is the reason for his elevated LFTs? Is there a specific marker for Alcoholic Fatty Liver disease?

- Excessive alcohol consumption is known to elevate LFTs. Alcohol is a known hepatotoxin which effects liver functioning and there is no relation between the amount consumed and the stage of liver damage.

https://www.ncbi.nlm.nih.gov/pmc/articles/PMC4155359/ 

Sensitivity and specificity of biomarkers in detecting harmful or heavy alcohol consumption

Biomarker        AST     ALT      MCV    CDT     CDT + GGT     CDT + GGT + MCV

Sensitivity        47%-68%         32%-50%         45%-48%         63%-84%         83%-90%         88%

Specificity        80%-95%         87%-92%         52%-94%         92%-98%         95%-98%         95%

 Reference: https://www.ncbi.nlm.nih.gov/pmc/articles/PMC4155359/

-GGT and CDT are usually taken as specific markers for Alcoholic Fatty Liver Disease. 

 

4) What is the line of treatment in this patient?

-Treatment in this patient is: 

Investigations:


Treatment:





Case #3:

https://chennabhavana.blogspot.com/2021/05/general-medicine-case-discussion-1.html

1) What is the most probable diagnosis in this patient?

àDifferential Diagnosis:

·         Ruptured Liver Abscess.

·         Organized collection secondary to Hollow viscous Perforation.

·         Organized Intraperitoneal Hematoma.

·         Free fluid with internal echoes in Bilateral in the Subdiaphragmatic space.

·         Grade 3 RPD of right Kidney

àThe most probably diagnosis is there is abdominal hemorrhage. This will give reasoning to the abdominal distention, and the blood which is aspirated.

 

2) What was the cause of her death?

àAfter leaving the hospital, the patient went to Hyderabad and underwent an emergency laparotomy surgery. The patient passed away the next day. Cause of her death can be due to complications of laparotomy surgery such as, hemorrhage (bleeding), infection, or damage to internal organs.

https://www.ncbi.nlm.nih.gov/pmc/articles/PMC5573610/#:~:text=Conclusion,SBP%20%E2%89%A4%2090%20mmHg%20dying.

 

3) Does her NSAID abuse have something to do with her condition? How? 

à https://www.ncbi.nlm.nih.gov/pmc/articles/PMC3158445/

àNSAID-induced renal dysfunction has a wide spectrum of negative effects, including decreased glomerular perfusion, decreased glomerular filtration rate, and acute renal failure. Chronic NSAIDs use has also been related to hepatotoxicity. While the major adverse effects of NSAIDs such as gastrointestinal mucosa injury are well known, NSAIDs have also been associated with hepatic side effects ranging from asymptomatic elevations in serum aminotransferase levels and hepatitis with jaundice to fulminant liver failure and death. 


5) NEPHROLOGY:

Case #1:

https://kavyasamudrala.blogspot.com/2021/05/medicine-case-discussion-this-is-online.html

1) What could be the reason for his SOB?

- About 2% of the body's creatine, is converted into creatinine every day and is transported to the kidneys for excretion. The kidneys function to eliminate most of the creatinine through urine. A rise in creatinine is seen when there is kidney disease. 

-This patient underwent a TURP around two months ago, post which he came back to the hospital with hyponatremia, and raised creatinine levels. 

- Normal levels of creatinine in the blood are 0.6 to 1.2 mg/dl in adult males and 0.5 to 1.1 mg/dl in adult females. 

- After discharge, he came for routine testing about 1 week later, with elevated creatinine levels (6.2 mg/dl), and then presented to the hospital with SOB on exertion with a serum creatinine level raised to 10 mg/dl. 

-This increase in serum creatinine could have been the cause of shortness of breath in the patient.

 

2) Why does he have intermittent episodes of drowsiness?

- The intermittent episodes of drowsiness can be caused by hyponatremia. 

-Hyponatremia is defined as when the concentration of sodium in your blood is abnormally low. Sodium is an electrolyte, and it helps regulate the amount of water that's in and around your cells.

- In hyponatremia, some factors ranging from an underlying medical condition to drinking too much water can cause the sodium in your body to become diluted. When this happens, your body's water levels rise, and your cells begin to swell. 

-Hyponatremia signs and symptoms seen in this case are: 


-In this case, post TURP in the patient he presented to the hospital with drowsiness and excessive sleep that attenders felt difficult to wake him up from sleep.  His attenders were regularly monitoring his BP, which was found to be fluctuating and patient was brought to the hospital and was found with hyponatremia, which is what may have caused his drowsiness.


3) Why did he complaint of fleshy mass like passage in his urine?

- The patient in this case has a history of dysuria. His urine is cloudy in appearance.

- On investigation, there is a presence of pus cells in the urine. The patient has an indication of hydronephrosis.

-Hydronephrosis is a condition that typically occurs when the kidney swells due to urine failing to properly drain from the kidney to the bladder. Due to stasis of fluid, these patients become predisposed to development of urinary tract infection, which can be the cause of fleshy type masses or gritty particles in the urine.


4) What are the complications of TURP that he may have had?

-The complications of TURP he might have had are: 




Case #2:

https://drsaranyaroshni.blogspot.com/2021/05/an-eight-year-old-with-frequent.html

1) Why is the child excessively hyperactive without much of social etiquettes?

- According to the case history, in this case an 8 year old boy, is excessively hyperactive, impulsive, does not have proper social etiquettes as is expected of his age, too active to pay any attention at school, and he talks so fast that even comprehending sentences becomes quite difficult. 

-These issues are prominent enough to be negatively affecting his daily life. A physician should consider ADHD, a mental health disorder that can cause above-normal levels of hyperactive and impulsive behaviors. People with ADHD may also have trouble focusing their attention on a single task or sitting still for long periods of time.

- People who have ADHD have combinations of these symptoms:

Signs of hyperactivity and impulsivity are: 


(Reference link- https://www.nimh.nih.gov/health/publications/attention-deficit-hyperactivity-disorder-adhd-the-basics/)


2) Why doesn't the child have the excessive urge of urination at night time?

i. They have done a complete urine exam, a urine culture and sensitivity test, and checked levels of serum electrolytes. The cell counts in the urine examination are normal, and there are no microorganisms after urine culture. There is the presence of some mildly enlarged mesenteric lymph nodes on USG, there still remains suspicion of an infective etiology, including cox bacilli.

ii. Since the child does not get the urgency to urinate when he is asleep, there can be a chance of the manifestation being psychosomatic, or as a result of an undiagnosed anxiety disorder, or a stressor triggering this manifestation. 

iii. From the history of excessive hyperactivity, impulsiveness, lacking of attentivity, a thought goes towards Attention Deficit Hyperactivity Disorder and in turn towards the association urination disorders. Studies indicate that children with ADHD have a higher tendency to lack bladder control, and are prompted to go to the bathroom more frequently. 

-When you feel anxious, your body’s fear response can be triggered, overwhelming your bladder’s mechanisms for retaining urine, causing you to want to urinate. The child may be free from the triggers while asleep, especially because he does not have a history of nocturnal enuresis, thus enabling him to sleep well through the night.


3) How would you want to manage the patient to relieve him of his symptoms?

-The diagnosis of the child is leaning towards the possibility of a psychosomatic overactive bladder, which can be triggered by various stressors or the possibility of the child having undiagnosed ADHD, a conclusion based on history of his behavior. 

i. The most common treatment options include bladder retraining and pelvic floor exercises.

-Bladder retraining involves putting the child on a “voiding schedule” where they go to the restroom to urinate on a schedule. This helps to slowly train the bladder to hold more and more urine, as it is designed to. Pelvic floor exercises provide a way to strengthen the muscles that are used to slow and stop the flow of urine and prevent wetting.

ii. If these techniques do not work or seem to have no effect on the overactive bladder, there are some medications that can be used to calm the overactive bladder. Drugs like oxybutynin can help in controlling an overactive bladder. 

iii. Along with these, certain lifestyle measures can be adopted to help with an overactive bladder.

-Children with ADHD welcome easily accessible distractions. By decreasing time with electronics and increasing time doing engaging activities outside the home, your child will have an outlet for built-up energy. Get regular, daily physical activity and exercise. This can help manage the stressors that can trigger an overactive bladder in children.


6) INFECTIOUS DISEASES (HI virus, Mycobacteria, Gastroenterology, Pulmonology):

Case #1:

https://vyshnavikonakalla.blogspot.com/2021/05/a-40-year-old-lady-with-dysphagia-fever.html

1.Which clinical history and physical findings are characteristic of tracheoesophageal fistula?

àIn adults, the characteristic history is recurrent pneumonia, hemoptysis, and coughing after eating. In children, the features found in cases of congenital tracheoesophageal fistula, there is Frothy, white bubbles in the mouth, coughing or choking when feeding, vomiting, blue color of the skin (cyanosis), especially when the baby is feeding, difficulty breathing.

à The physical findings are, in the presence of TEF, abdominal distention may occur secondary to collection of air in the stomach.

2. What are the chances of this patient developing immune reconstitution inflammatory syndrome? Can we prevent it? 

à Immune reconstitution inflammatory syndrome (IRIS) is a condition seen in some cases of AIDS or immunosuppression, in which the immune system begins to recover, but then responds to a previously acquired opportunistic infection with an overwhelming inflammatory response that paradoxically makes the symptoms of infection worse.

àThere are chances that this patient can develop IRIS due to the patient being RVD positive. She is more susceptible to any infection and therefor prone to reinfection. To prevent IRIS, the most effective method is to involve the initiation of ART before immunosuppression is advanced. IRIS is uncommon in individuals who initiate antiretroviral treatment with a CD4+ T-cell count greater than 100 cells/uL. Aggressive efforts should be made to detect asymptomatic mycobacterial or cryptococcal disease prior to the initiation of ART, especially in areas endemic for these pathogens and with CD4 T-cell counts less than 100 cells/uL.

https://www.infectiousdiseaseadvisor.com/home/decision-support-in-medicine/infectious-diseases/immune-reconstitution-inflammatory-syndrome/#:~:text=How%20can%20immune%20reconstitution%20inflammatory,greater%20than%20100%20cells%2FuL

7) INFECTIOUS DISEASES AND HEPATOLOGY

Case #1:

https://kavyasamudrala.blogspot.com/2021/05/liver-abscess.html

1) Do you think drinking locally made alcohol caused liver abscess in this patient due to predisposing factors present in it ? What could be the cause in this patient ?

-Patient is toddy drinker since for the past 30 years and by occupation he is a palm tree climber. Toddy is a locally brewed beverage, when the conditions are unhygienic it gets contaminated with bacteria, fungi, and parasites.

-The most common contamination is Entamoeba Histolytica. This is a causative agent for liver abscess. 

- Poor nutrition can lead to the extended survival of the parasite in the host. This environment is aided in situations where the patient belongs to a lower socio-economic group. 


2) What is the etiopathogenesis  of liver abscess in a chronic alcoholic patient ? ( since 30 years - 1 bottle per day)

- Alcohol causes Amoebic Liver Abscess. This occurs through a multitude of mechanisms:


3) Is liver abscess more common in right lobe?

-Liver abscess is more common in right lobe than left lobe. The involvement of right lobe to left lobe is in the ratio of 2: 1. 

-This occurs due to:


4) What are the indications for ultrasound guided aspiration of liver abscess ?

-Indications for ultrasound guided aspiration of a liver abscess are:


Case #2:

https://63konakanchihyndavi.blogspot.com/2021/05/case-discussion-on-liver-abcess.html

1) Cause of liver abscess in this patient?

-Patient is an occasional Toddy drinker. Toddy is a local alcoholic beverage when collected in unhygienic environment, might get contaminated with pathogens such as bacteria, fungi and parasites.

-These pathogens travel through the portal circulation reach the liver and might result in liver abscess formation in the patient.

2) How do you approach this patient?

-When patient presents with the symptoms of this patient such as pain and fever, the approach taken is:


  -Treatment should be given accordingly. 

    =In practice an empirical treatment is given to treat both amoebic and pyogenic liver abscess -

    =This includes use of Broad spectrum antibiotics and Metronidazole 

    =Analgesics and anti inflammatory drugs -to relieve pain and fever.

    =Multivitamin supplements

    =Saline infusion- to maintain fluid levels. 

-All the above medicines should be given for  7- 10 days.

-Following this review the patient and see if there is any improvement. USG abdomen should be done se if the abscess is resolving. If there is no improvement then USG guided aspiration should be done.

    


3) Why do we treat here ; both amoebic and pyogenic liver Abscess? 

-The presentation for both amoebic , pyogenic liver abscess is the same. They both present with pain abdomen, fever, constitutional symptoms like nausea and vomiting, and loss of appetite. 

-The investigations that are done are:

    =USG: a hypo echoic mass for both type of abscess.

    =Amoebic and pyogenic liver abscess can be differentiated only by culture and sensitivity of the aspirate obtained       by USG guided aspiration.

-USG guided aspiration has the following risk factors:

1) If abscess is thin walled there is a risk of rupture.

2) If abscess is on the posterior aspect of the liver, it will not be accessible.

3) There is also a risk of bleeding.

-There is leucocytosis, and elevated alkaline phosphatase, ALT, AST.

- Blood culture taken prior to the administration of antibiotics is helpful for identifying the causative organism but  as this patient had already taken antimicrobials before he came to the hospital, there is severe abdominal pain treatment is started immediately without a blood culture report.

-We treat both forms of Liver abscess empirically using-

    = Broad spectrum antibiotics: a combination of penicillin, cephalosporin, aminoglycosides.

    = Metronidazole: has both antibacterial and antiprotozoal activity.

-Reference: https://academic.oup.com 


4) Is there a way to confirm the definitive diagnosis in this patient?

-Liver abscess can be confirmed by USG Abdomen 

- It presents as single/multiple, round/oval, hypoechoic/hyperechoic mass more commonly is the right lobe of the liver. However USG cannot differentiate an amoebic liver abscess from pyogenic liver abscess.

-For this 

    = Blood culture 

    = USG guided aspiration of the abscess should be done.

-This aspirate should be subjected to antigen testing for: 

-Subjected to microbiological culture and sensitivity helps to identify pyogenic organisms.

8) INFECTIOUS DISEASES (Mucormycosis, Ophthalmology, Otorhinolaryngology, Neurology):

Case #1:

http://manikaraovinay.blogspot.com/2021/05/50male-came-in-altered-sensorium.html 

1) What is the evolution of the symptomatology in this patient in terms of an event timeline and where is the anatomical localization for the problem and what is the primary etiology of the patient's problem?

3 years ago

Diagnosed with HTN

21 days ago

Vaccination followed by fever with chills and rigors, relieved on medication.

18 days ago

Similar complains, went to hospital, not relieved on medication.

11 days ago

Generalized weakness and facial puffiness and periorbital edema

4 days ago

In altered state with facial puffiness and periorbital edema and weakness of right upper limb and lower limb. Towards the evening patient periorbital edema progressed. Serous discharge from the left eye that was blood tinged. Patient was diagnosed with diabetes mellitus.

Patient was referred to a government general hospital

 

2 days ao

Patient passed away.

Anatomical localization:

-The patient was also diagnosed with acute infarct in the left frontal and temporal lobe.

Etiology:

Patient was diagnosed with diabetic ketoacidosis and was unaware that he was diabetic until then. This resulted in poorly controlled blood sugar levels. The patient was diagnosed with acute ororhino orbital mucormycosis. Rhino cerebral mucormycosis is the most common form of this fungus that occurs in people with uncontrolled diabetes the fungus enters the sinuses from the environment and then the brain.

-Reference: https://www.cdc.gov/fungal/diseases/mucormycosis/definition.html

- Mucormycosis is associated with the occurrence of CVA.

-Reference: https://journal.chestnet.org/article/S0012-3692(19)33482-8/fulltext#:~:text=There%20are%20few%20incidences%20reported,to%20better%20morbidity%2Fmortality%20outcomes.


2) What is the efficacy of drugs used along with other non-pharmacological treatment modalities and how would you approach this patient as a treating physician?

The proposed management of the patient was – 

1. Inj. Liposomal amphotericin B according to creatinine clearance 

2. 200mg Itraconazole was given as it was the only available drug which was adjusted to his creatinine clearance

3. Deoxycholate was the required drug which was unavailable

-Reference: https://pubmed.ncbi.nlm.nih.gov/23729001/

- This article talks about the efficacy and toxicity of different formulations of amphotericin B. Along with the above mentioned treatment for the patient managing others symptoms is also done by-

- Management of diabetic ketoacidosis – 

(a) Fluid replacement-  The fluids will replace those lost through excessive urination, as well as help dilute the excess sugar in blood.

(b) Electrolyte replacement-The absence of insulin can lower the level of several electrolytes in blood. Patient will receive electrolytes through a vein to help keep the heart, muscles and nerve cells functioning normally.

(c) Insulin therapy-  Insulin reverses the processes that cause diabetic ketoacidosis. In addition to fluids and electrolytes, patient will receive insulin therapy

3) What are the postulated reasons for a sudden apparent rise in the incidence of Mucormycosis in India at this point of time? 

-Mucormycosis is may be being triggered by the use of steroids.

- It is a life-saving treatment for severe and critically ill Covid-19 patients. Steroids reduce inflammation in the lungs for Covid-19 and appear to help stop some of the damage that can happen when the body's immune system goes into overdrive to fight off coronavirus. 

-The steroid can cause immunosuppression and can lead to an increase in the blood sugar level in patients with diabetes and without. 

-This immunosuppression leads to an increase in the incidence of Mucormycosis. 


9) INFECTIOUS DISEASES - COVID-19

Link to Master Case Sheet:

https://drive.google.com/file/d/1BBHfmdO5JDN7UVLNWLX8o_ykwN8e-6qZ/view?usp=sharing 


10) Medical Education:

-Throughout the month of May, I have attended the corresponding 4th year classes in the 8-10 sessions during the week (Mon-Sat). 

-Followed by the third year practicals from 2-3 everyday in the afternoon. On the days where this class is cancelled, I joined the integrated medicine sessions that happen from 2-4, where we can listen to the presentation of the cases.

-While making my first e-log, I experienced taking telephonic history for the first time, followed by making a detailed chart of the patient. This gave me an indepth view into the constituents of a covid case. 

-With he bimonthly assessment, I was able to read through a multitude of different scenarios and understand the etiology and treatment of the cases. The assessment made me branch out and read many trials and journals that ascertain the link between different presentations. 

























Comments

Popular posts from this blog

INTERNSHIP ASSESSMENT

45F SYSTEMIC LUPUS ERYTHEMATOSIS